4
$\begingroup$

I'm reading a proof of below theorem from this paper.

Theorem A.3. Let $\Omega$ be a locally compact normal Hausdorff space. Let $\left\{\mu_n\right\} \cup\{\mu\} \subset \mathcal{M}(\Omega)$ and assume that $\underset{n \rightarrow \infty}{\operatorname{v-lim}} \mu_n=\mu$. Then for any open set $\Theta \subset \Omega$, $$ |\mu|(\Theta) \leq \liminf _{n \rightarrow \infty}\left|\mu_n\right|(\Theta) . $$ In particular, $\|\mu\| \leq \liminf _{n \rightarrow \infty}\left\|\mu_n\right\|$.

Proof. Let $\Theta \subset \Omega$ be open and $\varepsilon>0$. Since $\mu$ is inner regular and $\Omega$ is normal and locally compact, as a consequence of Urysohn's lemma [1, Lemma 2.46], there exists $f \in C_c(\Omega)$ such that $|f| \leq 1, \operatorname{supp}(f) \subset \Theta$ and $$ \int f \mathrm{~d} \color{red}{\mu} \geq \color{red}{|\mu|}(\Theta)-\varepsilon. $$ Then by vague convergence of $\left\{\mu_n\right\}$, $$ |\mu|(\Theta)-\varepsilon \leq \int f \mathrm{~d} \mu=\lim _{n \rightarrow \infty} \int f \mathrm{~d} \mu_n \leq \liminf _{n \rightarrow \infty} \int|f| \mathrm{d}\left|\mu_n\right| \leq \liminf _{n \rightarrow \infty}\left|\mu_n\right|(\Theta) $$ Now the result follows by letting $\varepsilon \downarrow 0$.


My understading: Below are the authors' related definitions. A finite signed measure $\mu$ is Radon if its variation $|\mu|$ is inner regular. In above proof, $\mu$ is Radon and thus $|\mu|$ is inner regular. So there is $f \in C_c(\Omega)$ such that $f \le 1_\Theta$ and $$ \int f \mathrm{~d} \color{red}{|\mu|} \geq \color{red}{|\mu|}(\Theta)-\varepsilon. $$

So I'm confused due to the appearance of $\color{red}{\mu}$ instead of $\color{red}{|\mu|}$ in the integral. Could you elaborate on my confusion?


Authors' definitions: Let

  • $\Omega$ be a metric space and $\mathscr{B}(\Omega)$ its Borel $\sigma$-algebra.
  • $C_b(\Omega)$ the subspace of all real-valued bounded continuous functions on $\Omega$.
  • $C_0(\Omega)$ the subspace of all $f \in C_b(\Omega)$ such that for any $\varepsilon>0$, there exists a compact set $K_{\varepsilon}$ with $|f|<\varepsilon$ on $K_{\varepsilon}^c$, and
  • $C_c(\Omega)$ the subspace of all $f \in C_b(\Omega)$ such that $f$ has compact support.

For a signed measure $\mu$ on $(\Omega, \mathscr{B}(\Omega)$ ), we denote by $|\mu|$ its associated variation measure. A finite signed measure $\mu$ on $(\Omega, \mathscr{B}(\Omega))$ is called a finite signed Radon measure if $|\mu|$ is inner regular, i.e., for each $A \in \mathscr{B}(\Omega)$, $$ |\mu|(A)=\sup \{|\mu|(K): K \in \mathscr{B}(\Omega), K \text { compact, } K \subset A\} . $$ We denote the set of all finite signed Radon measures on $(\Omega, \mathscr{B}(\Omega))$ by $\mathcal{M}(\Omega)$ and the subset of all finite positive Radon measures by $\mathcal{M}^{+}(\Omega)$. We say that a sequence $\left\{\mu_n\right\} \subset \mathcal{M}(\Omega)$ converges to $\mu \in \mathcal{M}(\Omega)$

  • (a) weakly if $\int_\Omega f \mathrm d \mu_n \to \int_\Omega f \mathrm d \mu$ for all $f \in C_b(\Omega)$ and we write $\underset{n \rightarrow \infty}{\operatorname{w-lim}} \, \mu_n= \mu$;
  • (b) vaguely if $\int_\Omega f \mathrm d \mu_n \to \int_\Omega f \mathrm d \mu$ for all $f \in C_c(\Omega)$, and we write $\underset{n \rightarrow \infty}{\operatorname{v-lim}} \, \mu_n=\mu$.
$\endgroup$

2 Answers 2

4
$\begingroup$

$\newcommand{\Om}{\Omega}\newcommand{\Th}{\Theta}\newcommand{\B}{\mathscr B}\newcommand{\M}{\mathcal M}\newcommand\ep\varepsilon\newcommand{\de}{\delta}\newcommand{\R}{\mathbb R}$Take any $\mu\in\M(\Om)$, any open subset $\Th$ of $\Om$, and any real $\ep>0$. Let $\de:=\ep/4$.

By the Hahn decomposition theorem, there is a partition of $\Om$ into Borel sets $D^\pm$ such that $D^+$ is a positive set for $\mu$ and $D^-$ is a negative set for $\mu$.

Let \begin{equation*} A^\pm:=\Th\cap D^\pm. \tag{1}\label{1} \end{equation*} Since $|\mu|$ is inner regular, there exist compact sets \begin{equation*} K^\pm\subseteq A^\pm\text{ such that }|\mu|(A^\pm\setminus K^\pm)<\de. \tag{2}\label{2} \end{equation*} Since $\Om$ is normal, there exist open subsets $U^\pm$ of $\Th$ such that \begin{equation*} U^\pm\supseteq K^\pm\text{ and }U^+\cap U^-=\emptyset. \tag{3}\label{3} \end{equation*} Since the sets $K^\pm$ are compact and $\Om$ is locally compact, without loss of generality the closures of the sets $U^\pm$ are compact.

By Urysohn'slemma, there exist continuous functions $f^\pm\colon\Om\to\R$ such that \begin{equation*} 0\le f^\pm\le1,\quad f^\pm=1\text{ on }K^\pm,\quad f^\pm=0\text{ on }\Om\setminus U^\pm. \tag{4}\label{4} \end{equation*} Let \begin{equation*} f:=f^+-f^-. \end{equation*} Then $f^+f^-=0$, whence $|f|\le1$. Also, $f=0$ on $\Om\setminus(U^+\cup U^-)$. So, recalling that the closures of the sets $U^\pm$ are compact, we see that $f\in C_c(\Om)$. Also, since $U^\pm$ are subsets of $\Th$, we have $|f|\le1_\Th$.

It remains to show that \begin{equation*} \int_\Om f\,d\mu\ge|\mu|(\Th)-\ep. \tag{$*$}\label{*} \end{equation*} To do this, note that, by \eqref{3}, \eqref{2}, and \eqref{1},
\begin{equation} \begin{aligned} |\mu|(U^-\setminus K^-)&\le|\mu|(\Th\setminus U^+\setminus K^-) \\ &=|\mu|(\Th)-|\mu|(U^+)-|\mu|(K^-) \\ &\le|\mu|(\Th)-|\mu|(K^+)-|\mu|(K^-) \\ &<|\mu|(\Th)-|\mu|(A^+)-|\mu|(A^-)+2\de=2\de. \end{aligned} \tag{5}\label{5} \end{equation} So, by \eqref{4}, \eqref{3}, \eqref{2}, \eqref{5}, and \eqref{1}, \begin{equation*} \begin{aligned} \int_\Om f\,d\mu&=\int_{U^+} f^+\,d\mu-\int_{U^-} f^-\,d\mu \\ &\ge\int_{K^+} f^+\,d\mu-\int_{K^-} f^-\,d\mu -\int_{U^-\setminus K^-} f^-\,d\mu \\ &=\mu(K^+)-\mu(K^-) -\int_{U^-\setminus K^-} f^-\,d\mu \\ &\ge\mu(K^+)-\mu(K^-) -|\mu|(U^-\setminus K^-) \\ &>\mu(A^+)-\de-\mu(A^-)-\de -2\de \\ &=|\mu|(\Th)-4\de=|\mu|(\Th)-\ep, \end{aligned} \end{equation*} so that \eqref{*} is proved. $\quad\Box$

$\endgroup$
5
  • $\begingroup$ May I ask if it should be "$\ge \mu(K^+)-\mu(K^-) -\int_{U^-\setminus K^-} f^-\,d\mu$" rather than "$=\mu(K^+)-\mu(K^-) -\int_{U^-\setminus K^-} f^-\,d\mu$" in the transformation at the end? $\endgroup$
    – Analyst
    Nov 6, 2022 at 3:04
  • $\begingroup$ @Analyst : The equality there holds because $f^\pm=1$ on $K^\pm$. So, of course, the inequality $\ge$ also holds. $\endgroup$ Nov 6, 2022 at 14:12
  • $\begingroup$ But we only know that $f^\pm (\Omega \setminus U^\pm) =0$ so it's possible that $f^\pm >0$ on $U^\pm \setminus K^\pm$. Maybe I'm wrong... $\endgroup$
    – Analyst
    Nov 6, 2022 at 14:18
  • 1
    $\begingroup$ @Analyst : I don't think your latter comment is wrong. But what is your question now, if any? $\endgroup$ Nov 6, 2022 at 15:30
  • $\begingroup$ Could you please have a look at this related question? Thank you so much for your detailed answer here. $\endgroup$
    – Analyst
    Nov 6, 2022 at 15:33
2
$\begingroup$

First we state here some general results about Radon measures following Bichteler, K. Integration: A Functional Approach, Birkhauser, 1991 and Bichteler, K. Integration theory: With Special Attention to Vector Measures, Lecture Notes in Mathematics, Springer 1973.

Let $\mathcal{K}$ denote the collection of compact subsets in $\Omega$ and $\mathcal{G}$ the collection of all open subsets in $\Omega$.

The Markov-Riesz representation theorem states that if $\Omega$ is a l.c. H. space and $I$ is a nonnegative linear functional on $\mathcal{C}_{00}(\Omega)$ then there is a order continuous and non-negative measure $\mu$ on a $\sigma$-algebra $\mathscr{M}(\Omega)$ containing the Borel sets such that

  • A. For any compact set $K\subset \Omega$ $$\mu(K)=\inf\{I(\phi): K\prec \phi\}<\infty$$

  • B. For any open set $G\subset \Omega$, $$\mu(G)=\sup\{I(\phi):0\leq \phi\prec G\}=\sup\{\mu(K): K \in\mathcal{K},\,K\subset G\}$$

  • C. For any $A\subset \Omega$, $$\mu^\bullet(A)=\inf\{\mu(G): G\in\mathcal{G},\, A\subset G\}$$

  • D. For any $A$ with $\mu(A)<\infty$, $$\mu(A)=\sup\{\mu(K): K\in\mathcal{K},\, K\subset A\}$$

Conditions B, D are called inner regularity; condition $C$ is called outer regularity.

The notation $K\prec f\prec G$, where $K\in\mathcal{K}$, $f\in\mathcal{C}_{00}(X)$, $G\in\mathcal{G}$ means that $\mathbb{1}_K\leq f\leq \mathbb{1}_G$, $\operatorname{supp}(f)\in\mathcal{K}$ and $\operatorname{supp}\subset G$.

A real (or complex) Radon measure $m$ is a linear functional on $\mathcal{C}_{00}(\Omega)$ that satisfies the property:
Property R: For any sequence $(\phi_n:n\in\mathbb{N})\subset\mathcal{C}_{00}(\Omega)$ whose supports are contained in a common compact set and which converges uniformly to some $\phi\in\mathcal{C}_{00}(\Omega)$, $\lim_n m(\phi_n)=m(\phi)$.

It is well known that

Theorem: A real linear functional on $\mathcal{C}_{00}(\Omega)$ satisfies property R iff and only if and $m$ has finite variation $|m|$, that is, for any $\psi\in\mathcal{C}^+_{00}(\Omega)$, \begin{align}|m|(\psi):=\sup\{m(\phi): \phi\in\mathcal{C}_{00}(\Omega), |\phi|\leq\psi\}<\infty\tag{1}\label{one}\end{align} The functional $|m|$ is linear and positive homogeneous on $\mathcal{C}_{00}^+(\Omega)$ and thus can be extended uniquely as a nonnegative linear functional on $\mathcal{C}_{00}(\Omega)$.

It follows from the Markov-rise representation theorem that if $m$ satisfies property R, then $|m|$ is a Radon measure in the sense that appears in many standard textbooks.

For any pair of measures $m$ and $n$ of finite variation, define $m\wedge n=\frac{m+n-|m-n|}{2}$ and $m\vee n=\frac{m+n+|m-n|}{2}$. $m\wedge n$ is the largest of all measures $\rho$ such that $\rho\leq m$ and $\rho\leq n$; similarly, $m\vee n$ is the smallest of all measures $\eta$ such that $m\leq \eta$ and $n\leq \eta$. Two measures $m$ and $n$ are orthogonal (mutually singular) iff $m\wedge n=0$.

For any real measure $m$ if finite variation, the measures $m_+=\frac{|m|+m}{2}$ and $m_-=\frac{|m|-m}{2}$ are orthogonal. Then the Hanh-Jordan decomposition theorem states that

Theorem: There is a measurable set $P$ such that \begin{align} m_+(\cdot)=m(\cdot\cap P)=|m|(\cdot \cap P),\qquad m_-(\cdot)=-m(\cdot\setminus P)=|m|(\cdot\setminus P)\tag{2}\label{two} \end{align}

For the rest of this posting, $\mathcal{M}(\Omega)$ denotes the space of (real) Radon measures $m$ that have finite total variation: $|m|(\Omega)<\infty$.


We now prove that under the setting of the OP, for any open set $G$ in $\Omega$ and $\varepsilon>0$ there is $f\in\mathcal{C}_{00}(\Omega)$ with $|f|\prec G$ and such that $$\int f\,d\mu>|\mu|(G)-\varepsilon$$

  • A short proof can be obtained from the Markov-Riesz representation theorem and the definition of variation \eqref{one}: $|\mu|$ is a a Radon measure and so, for any $\varepsilon>0$ there is $g\in\mathcal{C}_{00}(\Omega)$ with $0\leq g\prec G$ such that $|\mu|(\Omega)-\varepsilon/2<|\mu|(g)=\int g\,d|\mu|$. By \eqref{one}, there is $f\in\mathcal{C}_{00}(\Omega)$ such that $|f|\leq g$ and $|\mu|(g)-\varepsilon/2<\mu(f)$. Putting things toguether $$|\mu|(\Omega)-\varepsilon<\mu(f).$$

  • An alternative approach is based on the Hahn-Jordan decomposition theorem: The liner operators $\mu_+=\frac{|\mu|+\mu}{2}$ and $\mu_-=\frac{|\mu|-\mu}{2}$ are positive linear functionals on $\mathbb{C}_{00}$ and so by the Markov-Riesz representation theorem they extend to measure satisfying the conditions in the bullets above. Since $\mu_+$ and $\mu_-$ are orthogonal, there are $\mathcal{M}(X)$ measurable sets $P$ and $N=X\setminus P$ such that $\mu_+(N)=0=\mu_-(P)$. Given $\varepsilon>0$ there are compact sets $K_p\subset P\cap G$ and $K_n\subset N\cap G$ such that \begin{align} \mu_+(G_p)=\mu_+(P\cap G)<\mu_+(K_p) + \varepsilon/4,&\qquad \mu_-(G)=\mu_-(N\cap G)<\mu_-(K_n)+\varepsilon/4 \end{align} Further, since $K_p$ and $K_n$ are disjoint compact sets, and $\mu_\pm$ are Radon, there are disjoint open sets $G_p$, and $G_n$ such that $K_p\subset G_p\subset G$ and $K_n\subset G_n\subset G$. Uryshohn's lemma yields function $f_p,f_n\in\mathcal{C}_{00}(\Omega)$ such that $$K_p\prec f_p\prec G_p,\qquad K_n\prec f_n\prec G_n$$ Then \begin{align} \int(f_p-f_n)\,d\mu&=\int f_p\,d\mu_+ -\int f_p\,d\mu_- -\int f_n\,d\mu_+ +\int f_n\,d\mu_-\\ &\geq \mu_+(K_p) + \mu_-(K_n)-\mu_-(G_p)-\mu_+(G_n)\\ &\geq \mu_+(G)-\varepsilon/4+ \mu_-(G)-\varepsilon/4-\mu_-(G_p)-\mu_+(G_n)\\ &=|\mu|(G)- \varepsilon/2-\mu_-(G_p)-\mu_+(G_n) \end{align} Since $K_p\subset P$ \begin{align} \mu_-(G_p)&= \mu_-(G_p\setminus K_p)\leq \mu_-((G\setminus G_n)\setminus K_p )=\mu_-(G)-(\mu_-(G_n)+\mu_-(K_p))\\ &= \mu_-(G)-\mu_-(G_n)\leq \mu_-(G) -\mu_-(K_n)<\varepsilon/2 \end{align} Similarly, since $K_n\subset N=\Omega\setminus P$ \begin{align} \mu_+(G_n)&= \mu_+(G_n\setminus K_n)\leq \mu_+((G\setminus G_p)\setminus K_n )=\mu_+(G)-(\mu_+(G_p)+\mu_+(K_n))\\ &= \mu_+(G)-\mu_+(G_p)\leq \mu_+(G) -\mu_+(K_p)<\varepsilon/2 \end{align} Putting things together we obtain that $$\int (f_p-f_n)\,d\mu\geq |\mu|(G)-\varepsilon$$ Since $G_p\cap G_n=\emptyset$, $f:f_p-f_n$ satisfies $f\in\mathcal{C}_{00}(\Omega)$, and $|f|\leq 1$.

$\endgroup$
1
  • $\begingroup$ Thank you so much for your dedicated support and detailed answers! $\endgroup$
    – Analyst
    Nov 7, 2022 at 5:35

Your Answer

By clicking “Post Your Answer”, you agree to our terms of service and acknowledge you have read our privacy policy.

Not the answer you're looking for? Browse other questions tagged or ask your own question.